nazu.s.shaikh
Thanks Received: 0
Elle Woods
Elle Woods
 
Posts: 53
Joined: April 27th, 2010
 
 
trophy
First Responder
 

Q9 - There is relatively little

by nazu.s.shaikh Mon Jun 14, 2010 9:13 pm

I'm having difficulty with this argument, mainly trying to understand it. What exactly are they trying to say here .. that companies that make carpets will only gain revenue by buying their competitors product or shares?
User avatar
 
ManhattanPrepLSAT1
Thanks Received: 1909
Atticus Finch
Atticus Finch
 
Posts: 2851
Joined: October 07th, 2009
 
This post thanked 1 time.
 
 

Re: Q9 - There is relatively little

by ManhattanPrepLSAT1 Mon Jun 14, 2010 10:30 pm

I like this question! This one is sort of tricky...

The argument says that because people buy carpet so infrequently, that it is really hard to increase people's consumption of carpet. So, since you they cannot grow the market for carpet, in order to gain market share, companies will need to acquire their competitors.

We are asked to undermine the argument. To do this we need to show how a carpet producer could gain market share without purchasing a competitor. Answer choice (D) shows us how a carpet producer could do this. If price reductions force competitors to drop out of the business altogether, then whatever market share that company had before it went under would be acquired by a competitor.

(A) is out of scope. Other floor coverings are irrelevant to the market for carpet.
(B) strengthens the conclusion. If there is no remaining niche to fill, it becomes all the more likely that companies will have a difficult time growing their market share without purchasing a competitor.
(C) is out of scope. What happens to profit levels is irrelevant to the size of a competitor's share of the carpet market.
(D) undermines the argument by finding a way for a company to grow in market share without acquiring one of their competitors.
(E) strengthens the argument. If consumers are resistant to new patterns and styles, then it is more difficult for a carpet producer to grow through innovative product designs.

Hopefully this helps put the argument into perspective. If not, let me know what needs further explanation!
 
nazu.s.shaikh
Thanks Received: 0
Elle Woods
Elle Woods
 
Posts: 53
Joined: April 27th, 2010
 
 
trophy
First Responder
 

Re: PT10, S4, Q9 - There is relatively little room

by nazu.s.shaikh Fri Jun 25, 2010 1:21 am

Ahhh, that makes sense. One thing I don't understand though is that when a question asks you to weaken or strengthen ( in this case weaken) wouldn't we want new information to be brought in to help undermine the argument?

Humor me, but why wouldn't profit levels undermine the argument? Why is it out of scope?
User avatar
 
ManhattanPrepLSAT1
Thanks Received: 1909
Atticus Finch
Atticus Finch
 
Posts: 2851
Joined: October 07th, 2009
 
 
 

Re: PT10, S4, Q9 - There is relatively little room

by ManhattanPrepLSAT1 Mon Jun 28, 2010 2:04 am

Profits would definitely be considered new information. And it's always a good idea to see how the answer choices affect the argument's conclusion.

Let's start with that. The conclusion of the argument is that in order to gain market share in the carpet market, carpet makers would need to purchase a competitor. We're asked to undermine this claim, so we need an answer that will suggest how carpet makers could increase their share of the carpet market without purchasing a competitor.

Answer choice (C) is definitely tempting as an answer choice. Especially since it brings up mergers. But merging companies do not really gain market share. They combine their respective shares. And profits does not really relate to either market share or the purchasing of competitors.

I just can't see how answer choice (C) undermines the conclusion, but I'd love to hear your argument! Let me know how you see this undermining the conclusion...
 
nazu.s.shaikh
Thanks Received: 0
Elle Woods
Elle Woods
 
Posts: 53
Joined: April 27th, 2010
 
 
trophy
First Responder
 

Re: PT10, S4, Q9 - There is relatively little room

by nazu.s.shaikh Mon Jun 28, 2010 11:51 pm

The way I reasoned this was the conclusion stated that in order to gain market share they would need to acquire their competitor. Answer C would weaken the argument by showing that in the past previous companies merging with a new one has shown not to work. Therefore companies buying their competitors would not benefit either party.

I assumed this would weaken the argument enough; it doesn't necessarily have to weaken the argument 100% does it? Just a variant?
User avatar
 
ManhattanPrepLSAT1
Thanks Received: 1909
Atticus Finch
Atticus Finch
 
Posts: 2851
Joined: October 07th, 2009
 
This post thanked 1 time.
 
 

Re: PT10, S4, Q9 - There is relatively little room

by ManhattanPrepLSAT1 Tue Jun 29, 2010 7:23 pm

I agree that answer choice (C) would show that companies buying their competitors will not always work. But the conclusion doesn't say that it will always work. Rather it states that in order to gain market share, not necessarily related to profits, a company would need to purchase their competitors.

Even if profits and market share were related, the conclusion states that purchasing competitors is a necessary condition of gaining market share, not a sufficient condition that would ensure that something good is going to happen.

So, while I agree with your understanding of answer choice (C), I guess I would disagree on it's value in undermining the conclusion of the argument.

To your other point about the degree to which an answer choice would need to weaken the conclusion. You're absolutely correct. To weaken you do not need to destroy. Anything that would make the conclusion less likely to be true would be the correct answer!
User avatar
 
WaltGrace1983
Thanks Received: 207
Atticus Finch
Atticus Finch
 
Posts: 837
Joined: March 30th, 2013
 
 
trophy
Most Thanked
trophy
Most Thankful
trophy
First Responder
 

Re: Q9 - There is relatively little

by WaltGrace1983 Thu May 15, 2014 3:11 pm

Something that hasn't been addressed is the (entirety of?) the core. Just so I am clear, the core actually looks like the following, right?

    Most who purchase carpet only do so twice
    (→)
    Carpet companies will be able to again market share only through purchasing competitors
    →
    There is little room for growth in the overall carpet market


It seems that there being only ONE way to gain market share (according to the flawed argument) would support the idea that there is "little room for growth" in the that market share. Is this correct?
 
christine.defenbaugh
Thanks Received: 585
Atticus Finch
Atticus Finch
 
Posts: 536
Joined: May 17th, 2013
 
This post thanked 1 time.
 
 

Re: Q9 - There is relatively little

by christine.defenbaugh Mon May 19, 2014 1:13 pm

I actually think you have it just slightly turned around, WaltGrace1983, and I think some key definitions might be tripping you up just a tad.

The "growth in the overall carpet market" would mean in increase in the total number of carpets being purchased. The fact that there isn't any room for that growth (i.e., the pie ain't getting bigger) isn't the final conclusion of the argument, but is rather being used to support another claim.

If you were thinking of 'growth' as 'growth of a particular company's market share', then I can see how you'd get it turned around and think that [only one way for growth] supported a conclusion of [little room for growth].

There are two essential ways for a company to grow their own market share: take part of someone else's slice of pie somehow, or make the entire pie bigger. The premises here are telling us that making the pie bigger can't happen (because people only buy carpet once or twice, and that's just not changing). So the core looks something like this:

    PREMISE: Overall carpet market isn't going to grow (because people only buy carpets once/twice
    CONCLUSION: ONLY WAY to increase market share is to purchase competitors


Since the pie really can't get bigger, the only way to increase market share is going to be to take part of someone else's slice of the existing pie. But there are tons of potential ways to do that. We might use aggressive marketing to lure customers away from a competitor, or we might sue competitors for frivolous things to drive them out of business. We might start horrible rumors about our competitors to make people shun them, or we might engage in aggressive price wars that cause competitors to just shut down. Any of those are potential ways that we might be able to take a competitors slice of the pie.

The argument, though, ignores all these possibilities and stubbornly concludes that the only possible way would be to purchase competitors.

Does that help a bit? Pay careful attention to the difference here between growth of the overall market and growth of some company's individual market share!
User avatar
 
WaltGrace1983
Thanks Received: 207
Atticus Finch
Atticus Finch
 
Posts: 837
Joined: March 30th, 2013
 
 
trophy
Most Thanked
trophy
Most Thankful
trophy
First Responder
 

Re: Q9 - There is relatively little

by WaltGrace1983 Tue May 27, 2014 12:54 pm

Ok then let me see if I got this correct.

    Most people only purchase carpet once or twice
    (→)
    The total size of market isn't going to increase
    →
    The only way to gain market share is through purchasing competitors


This argument basically gives us the fallacy of false alternatives. It is assuming that there are only two ways to grow and, because the size of the market share isn't going to get bigger, then it must be true that there is only one other way to gain market share: purchasing competitors. To weaken this claim, we would simply show that there are other ways to increase an individual company's market share.

(D) does this by showing that one can increase market share not ONLY by purchasing competitors but rather through alternative means: price reductions. (D) says that we don't need to purchase competitors, we can instead drive the competitors out of business.

(A) is out of scope because we don't care about other floorings.

(B) might actually strengthen by increasing the likelihood that the size of the pie isn't going to get bigger because there is "no remaining niche to fill." In other words, everyone who needs to be served is already being served.

(C) talks about merging with other competitors perhaps through purchasing. However, we don't really know if this involves purchasing and we don't really know how a decline in profits/revenues (a discussion of absolute variables) relates to the market share (a discussion of relative variables). You can decline in profits/revenues and gain market share just as you can increase in profits/revenues but loss market share.

(E) just gives us more reason to believe the premise yet we don't need to explain why the pie isn't going to get any bigger.
 
reyessandovalarturo
Thanks Received: 0
Vinny Gambini
Vinny Gambini
 
Posts: 1
Joined: July 25th, 2016
 
 
 

Re: Q9 - There is relatively little

by reyessandovalarturo Mon Jul 25, 2016 9:00 pm

This goes to Matt Sherman.

In my opinion Answer Choice (C) undermines the conclusion of the Stimulus to the same degree as Answer Choice (D), but does so in a more direct manner. Therefore, Answer Choice (C) should be correct.

Here is my reasoning:

The conclusion of the argument is that the only way for a company to increase it's market share is to buy it's competitors.

Answer Choice (D), which has traditionally been considered the correct answer, attempts to undermine this conclusion by showing that there are other ways to increase market share other than buying off your competition. The Answer Choice claims that cost-cutting allows for lower prices, which is, in turn, "causing other producers to leave the market altogether."

Let's look at the reasoning and underlying assumption behind this answer by asking the following question: Why would lower prices cause other producers to leave the market? When people examine this answer, they see that it claims that other producers are leaving the market and quickly assume that their market share is being assimilated by the companies that remain. This, however, is a leap in logic. There is no reason to believe that this alone would cause people to buy from them. The real weight of the answer comes from the claim about the lower prices. Because of the lower prices, people choose to buy from those companies which are using cost-cutting measures. And since "market share" is commonly defined as the percentage of a market's sales earned by a particular company, the Answer Choice concludes that a company can increase it's sales (and therefore market share) by a method other than the one specified by the Stimulus.

It can at this point be concluded that Answer Choice (D) meets the criteria of weakening the conclusion. It is also at this point that answer (C) is ignored as irrelevant because it speaks of "profits and revenue" rather than market share. However, this is unwise. As seen above from the discussion regarding the reasoning behind Answer Choice (D), that Answer assumes that a larger market share results from increased sales (produced by the lower prices). Answer Choice (C) is founded on the same reasoning: that is, that market share is defined as the percentage of a market's sales earned by a particular company. It is common knowledge that "profits and revenue" can only increase and decrease proportionally to sales. Therefore, Answer Choice (C) also meets the criteria of weakening the conclusion by showing that buying off your competition does not necessarily result in in a larger market share, a.k.a. sales, a.k.a. profits and revenue, but does so in far more direct a manner than Answer Choice (D).

So what do you guys think?
-R.S.
 
MarkR495
Thanks Received: 0
Vinny Gambini
Vinny Gambini
 
Posts: 9
Joined: November 20th, 2017
 
 
 

Re: Q9 - There is relatively little

by MarkR495 Tue Dec 05, 2017 1:21 am

reyessandovalarturo Wrote:This goes to Matt Sherman.

In my opinion Answer Choice (C) undermines the conclusion of the Stimulus to the same degree as Answer Choice (D), but does so in a more direct manner. Therefore, Answer Choice (C) should be correct.

Here is my reasoning:

The conclusion of the argument is that the only way for a company to increase it's market share is to buy it's competitors.

Answer Choice (D), which has traditionally been considered the correct answer, attempts to undermine this conclusion by showing that there are other ways to increase market share other than buying off your competition. The Answer Choice claims that cost-cutting allows for lower prices, which is, in turn, "causing other producers to leave the market altogether."

Let's look at the reasoning and underlying assumption behind this answer by asking the following question: Why would lower prices cause other producers to leave the market? When people examine this answer, they see that it claims that other producers are leaving the market and quickly assume that their market share is being assimilated by the companies that remain. This, however, is a leap in logic. There is no reason to believe that this alone would cause people to buy from them. The real weight of the answer comes from the claim about the lower prices. Because of the lower prices, people choose to buy from those companies which are using cost-cutting measures. And since "market share" is commonly defined as the percentage of a market's sales earned by a particular company, the Answer Choice concludes that a company can increase it's sales (and therefore market share) by a method other than the one specified by the Stimulus.

It can at this point be concluded that Answer Choice (D) meets the criteria of weakening the conclusion. It is also at this point that answer (C) is ignored as irrelevant because it speaks of "profits and revenue" rather than market share. However, this is unwise. As seen above from the discussion regarding the reasoning behind Answer Choice (D), that Answer assumes that a larger market share results from increased sales (produced by the lower prices). Answer Choice (C) is founded on the same reasoning: that is, that market share is defined as the percentage of a market's sales earned by a particular company. It is common knowledge that "profits and revenue" can only increase and decrease proportionally to sales. Therefore, Answer Choice (C) also meets the criteria of weakening the conclusion by showing that buying off your competition does not necessarily result in in a larger market share, a.k.a. sales, a.k.a. profits and revenue, but does so in far more direct a manner than Answer Choice (D).

So what do you guys think?
-R.S.



I think you're reading too much into it. The questions is asking for the answer choice that casts the MOST DOUBT on the conclusion. The conclusion is about companies gaining market share by purchasing other companies, and how it is the only way it can be achieved. (D), which you agree undermines the conclusion, is the best choice. It is an overlooked possibility of how companies can gain market share in the carpet market, without having to buy out their competition. This is the only piece of the carpet market that the argument is concerned about -- gaining of market share by purchasing competition.

(C) to me, sounds like it discusses a possible outcome of a company increasing their market share, and how it was unsuccessful. This doesn't necessarily mean they didn't still gain advantage in the total market share available, and hold onto it. For instance, the two largest carpet companies in the market merge, increasing their market share to 60% of the total carpet market. The country hits a recession the following quarter, and people cut back on costs; carpets taking a big hit and total sales decreasing. The total market sales decrease, as well as their companies sales, but their share in the market stays the same, and they still control 60% of it. Relative to the current market, they are still the market share leader, but unfortunately are not profiting from it.

This questions got me to answer (C) the first time around, but after looking at it it makes a lot of sense why D is the most viable answer.
 
JosephV
Thanks Received: 9
Jackie Chiles
Jackie Chiles
 
Posts: 38
Joined: July 26th, 2017
 
This post thanked 1 time.
 
 

Re: Q9 - There is relatively little

by JosephV Wed Jan 17, 2018 1:04 pm

I'd like to address two points in reyessandovalarturo's post above. They are both in his second-to-last paragraph:

reyessandovalarturo Wrote:It can at this point be concluded that Answer Choice (D) meets the criteria of weakening the conclusion. It is also at this point that answer (C) is ignored as irrelevant because it speaks of "profits and revenue" rather than market share. However, this is unwise. As seen above from the discussion regarding the reasoning behind Answer Choice (D), that Answer assumes that a larger market share results from increased sales (produced by the lower prices). Answer Choice (C) is founded on the same reasoning: that is, that market share is defined as the percentage of a market's sales earned by a particular company. It is common knowledge that "profits and revenue" can only increase and decrease proportionally to sales. Therefore, Answer Choice (C) also meets the criteria of weakening the conclusion by showing that buying off your competition does not necessarily result in in a larger market share, a.k.a. sales, a.k.a. profits and revenue, but does so in far more direct a manner than Answer Choice (D).


1. It is not necessarily true that (D) assumes that larger market share results from increased sales, in absolute terms (that is, last year firm X sold 10 million units, this year X sells 12 million) as you seem to imply. It could be that the entire market is in a bust and the carpet companies are not competing on who will make the most profit but who will suffer the least loss.

To illustrate: there are three firms in the carpet market. Two, say X and Y, lower their production costs and hence their prices, while firm Z does not. As a result firm Z has to leave the market at the end of 2017. During 2018 X and Y pick up Z’s slack. However, since the entire market is slowing down, what happens is that instead of losing 1 million sales each in 2018 over 2017, X and Y each lose only 500,000 sales. Have sales increased? No, they have gone down. Has the share of at least one the two remaining companies gone up? Definitely!

2. To me it is not so obvious that “profits and revenue” can only increase and decrease proportionally to sales. What if prices went up or down and the resultant loss/gain of sales were not adequate enough to offset the change in profits/revenue? It could be that prices went up by 10%, sales went down by 3% and yet profits/revenue increased by 8%. My point is, there is no need for proportionality.